What is the volume of a hemisphere with radius 3 ft? What is the volume of a hemisphere with diameter 13 cm?

Answers

Answer 1

The volume of a sphere is given by

[tex]V_s=\frac{4}{3}\pi r^3[/tex]

Since a hemisphere is half sphere, its volume is given by

[tex]\begin{gathered} V_{}=\frac{4}{6}\pi r^3 \\ \text{which is equivalent to} \\ V_{}=\frac{2}{3}\pi r^3 \end{gathered}[/tex]

where r is the radius.

Case a.

In this part r=3 ft, then by substituting this values into our last formula we get

[tex]V=\frac{2}{3}(3.1416)(3^3)[/tex]

which gives

[tex]V=56.55ft^3[/tex]

Case b.

In this part r=(13/2) cm, then by substituting this values into our last formula we get

[tex]V=\frac{2}{3}(3.1416)(6.5^3)[/tex]

which gives

[tex]V=287.59cm^3[/tex]


Related Questions

Lina picks a 4 digit number.
The number is more than 5000.
The number is odd.
The second digit is a prime number.

How many different possible numbers could Lina pick?

Answers

Answer:

1000

Step-by-step explanation:

The easiest way to go about this is by turning our number into ABCD.

A will be in the 1000th spot, B 100th, C 10th, and D 1st.

A has to be ≥ 5. So 5, 6, 7, 8, 9.

B has to be a prime number. 2, 3, 5, 7.

C is able to be any number it wants, 0, 1, 2, 3, 4, 5, 6, 7, 8, 9.

Since the number is odd, D has to be 1, 3, 5, 7, 9.

A has 5 different possibilities. B has 4, C has 10, and D has 5.
You'll then multiply all of these numbers for your answer.
5 * 4 * 10 * 5 = 1000

6. Find the are of the Rectangle
776
in
13
in

Answers

Answer:

about 3.09

Step-by-step explanation:

A = l  x w

A = 13/7 x 7/6

A = 1.857 x 1.666

A = 3.09

Hope it helps!

Solve for z in -3 < z-1 < 3.Give the result in the interval notation and graph on a number line

Answers

Answer:

(-2,4)

Explanation:

Given the inequality:

[tex]-3First, we add 1 to all parts of the inequality.[tex]\begin{gathered} -3+1We can represent this in interval notation as:[tex](-2,4)[/tex]

The solution set is graphed on the number line below:

help meeeeeeeeeeeeeee pleaseeeeeee

Answers

Answer: 19.6 feet

Step-by-step explanation:

Using the Pythagorean theorem,

[tex]x^2 +(x+6)^2 =48^2\\\\x^2 +x^2 +12x+36=2304\\\\2x^2 +12x-2268=0\\\\x^2 +6x-1134=0\\\\x=\frac{-6 \pm \sqrt{6^2 -4(1)(-1134)}}{2(1)}\\\\x \approx 30.8 \text{ } (x > 0)\\\\\implies x+(x+6) \approx 67.6\\\\\therefore (x+(x+6))-48 \approx 19.6[/tex]

ſ x - 5y = -9( 4x + 4y = - 12Step 1 of 2: Determine if the point (-4,1) lies on both of the lines in the system of equations by substituting theordered pair into both equations.

Answers

Given:

[tex]\begin{gathered} x-5y=-9\ldots\ldots(1) \\ 4x+4y=-12\ldots\text{.}\mathrm{}(2) \end{gathered}[/tex]

The given point is (-4,1)

Let's check the ordered pair (-4,1) in the first equation

[tex]\begin{gathered} x-5y=-9 \\ (-4)-5(1)=-9 \\ -4-5=-9 \\ -9=-9 \end{gathered}[/tex]

Hence, (-4,1) is a solution of the first equation x-5y=-9.

Now, let's check (-4,1) in the second equation.

[tex]\begin{gathered} 4x+4y=-12 \\ 4(-4)+4(1)=-12 \\ -16+4=-12 \\ -12=-12 \end{gathered}[/tex]

So, (-4,1) is a solution of the second equation 4x+4y=-12.

Hence, (-4,1) is a solution of both equation in the system, then it is a solution to the overall system.

Charlotte states that (43)3
can be rewritten as 218
. Which of the following explains how she is correct? Select all that apply.

Answers

The statement that explains why Charlotte's submission is correct is that (4³)³ simplifies to (4⁹) and (4⁹) simplifies to 2¹⁸

How to determine why Charlotte's submission is correct?

From the question, the expression is given as

(43)3

Rewrite the expression properly

So, we have the following representation

(4³)³

Apply the law of indices to rewrite the exponents in the expression properly

So, we have

(4³)³ = 4³*³

Next, we evaluate the products

So, we have the following notation

(4³)³ = 4⁹

Solving further, we need to express 4 as the square of 2

So, we have

(4³)³ = (2²)⁹

Apply the law of indices to rewrite the exponents in the expression properly

So, we have

(4³)³ = 2²*⁹

Next, we evaluate the products

So, we have the following notation

(4³)³ = 2¹⁸

Hence, Charlotte's expression is correct

Read more about expressions at

https://brainly.com/question/4344214

#SPJ1

Possible question

Charlotte states that (4³)³ can be rewritten as 2¹⁸. Which of the following explains how she is correct? Select all that apply.

1. Mr. Z's class has 18 boys and 22 girls. What is the ratio of girls to boys?

2. What is the ratio of boys to total students?

3. What is the ratio of girls to total students?

Answers

1 ))

22:18
simplified to
11:9

2 ))

18:40
simplified to
9:20

3 ))

22:40
simplified to
11:20

the product of 5 - 2i and i is

Answers

Answer:

5i + 2

Step-by-step explanation:

(5-2i)(i)

5i - 2i^2

i^2= -1

5i -2(-1)

= 5i + 2

Type the correct answer in each box. Use numerals instead of words. This graph represents a quadratic function. What is the function’s equation written in factored form and in vertex form? Graph shows upward parabola plotted on a coordinate plane. The parabola has vertex at (2, minus 8) with the left slope at (0, 0) and the right slope at (4, 0).

Answers

The function’s equation written in factored form and in vertex form are respectively;

f(x) = 2x(x - 4)

f(x) = 2(x - 2)² - 8

How to Interpret Quadratic Graphs?

The factored form of a quadratic function is;

f(x) = a(x - p)(x - q)

where:

p and q are the x-intercepts

a is a constant

Now, we are given x-intercepts as; (0, 0) and (4, 0)

Thus;

f(x) = a(x - 0)(x - 4)

f(x) = ax(x - 4)

To find a, substitute the given vertex (2, -8) into the equation and solve for a:

2a(2 - 4) = -8

-4a = -8

a = 2

Thus;

f(x) = 2x(x - 4)

The vertex form of a quadratic equation is;

f(x) = a(x - h)² + k

where:

(h, k) is the vertex

a is constant

Since vertex is (2, -8), then we have;

f(x) = a(x - 2)² - 8

Put the coordinate (0, 0) to find a;

0 = a(0 - 2)² - 8

4a = 8

a = 2

Thus;

f(x) = 2(x - 2)² - 8

Read more about Quadratic Graphs at; https://brainly.com/question/14477557

#SPJ1

help meeeeeeeeeeeeeeeeeeee pleaseee rnnnn rn!!!!

Answers

The length of short side x is 4.5 units, the length of short side x+5 is 9.5 units and the length of longest side is 10.5 units.

By Pythagorean theorem,

[tex]( Hypotenuse)^{2} = (Base)^{2}+ (Perpendicular)^{2}[/tex]

Let                base = x+5

     perpendicular = x

        Hypotenuse = x+6

                                    [tex](x+6)^{2} = x^{2} +(x+5)^{2}[/tex]

                           [tex]x^{2} +12x+36 = x^{2} +x^{2}+10x+25[/tex][tex]2x^{2} +10x+25-x^{2} -12x-36=0[/tex]

                             [tex]x^{2} -2x-11=0[/tex]

                                               [tex]x = \frac{-(-2) + \sqrt{4 - 4(1)(-11)} }{2*1}[/tex] or [tex]\frac{-(-2) - \sqrt{4 - 4(1)(-11)} }{2*1}[/tex]

                                               [tex]x=\frac{2 + \sqrt{4 +44} }{2}[/tex] or  [tex]\frac{2 - \sqrt{4 +44} }{2}[/tex]

                                               [tex]x = \frac{2 + \sqrt{48} }{2}[/tex] or [tex]\frac{2 - \sqrt{48} }{2}[/tex]

                                               [tex]x = \frac{2+2\sqrt{12} }{2}[/tex] or [tex]\frac{2-2\sqrt{12} }{2}[/tex]

                                               [tex]x = 1+\sqrt{12}[/tex] or [tex]1-\sqrt{12}[/tex]

                                               [tex]x = 1+3.5[/tex] or [tex]1-3.5[/tex]

                                               [tex]x = 4.5[/tex] or [tex]-2.5[/tex]

As, length can't be negative, we will take x = 4.5

Therefore, x = 4.5

            x + 5 = 4.5 + 5

                      = 9.5

             x + 6 = 4.5 + 6

                       = 10.5

Hence, the length of short side x is 4.5 units, the length of short side x+5 is 9.5 units and the length of longest side is 10.5 units.

Learn more about length on:

https://brainly.com/question/8552546

#SPJ1

while shopping for clothes, Daniel spent $26 less than 2 times what curtis spent. Daniel spent $10. write and solve an equation to find how much curtis spent. let x represent how much curtis spent

Answers

while shopping for clothes, Daniel spent $26 less than 2 times what curtis spent. Daniel spent $10. write and solve an equation to find how much curtis spent. let x represent how much curtis spent​

Let

x ------> amount that Curtis spent

we have that

10=2x-26 ------> equation that represent this situation

solve for x

2x=10+26

2x=36

x=$18

therefore

Curtis spent $18

Suppose that h(x) varies inversely with x and h(x)=50 when x = 0. 25. What is h(x) when x = 2?.

Answers

If h(x) varies inversely with x , then the value of h(x) is 6.25  , when x = 2 .

In the question ,

it is given that ,

the function h(x) varies inversely with x ,

that means h(x) ∝ 1/x ,

to remove the proportionality sign , we write the constant of proportionality  "k",

h(x) = k*(1/x)

given , that h(x)=50 when x = 0. 25

Substituting the values we get

50 = k*(1/0.25)

50 = k/0.25

k = 50×0.25

k = 12.5

So , the equation becomes

h(x) = 12.5/x

to find the value of h(x) when x = 2, we put  x = 2 , in the equation ,

h(2) = 12.5/2

= 6.25

Therefore ,  If h(x) varies inversely with x , then the value of h(x) is 6.25  , when x = 2 .

Learn more about Equations here

https://brainly.com/question/6995691

#SPJ4

solveeeeeeeeeeeee for x

Answers

Answer:

7.5/3

Step-by-step explanation:

set AC and BD equal to eachother because they are equal and solve

Brian had p pencils. Then he bought 4 more.
How many does he have now? due tm pleaseeeeeeeeeeeeee help

Answers

Answer: The number of pencils Brian has is p+4

Step-by-step explanation:

Because p is the original number of the pencil ( we don't know how many are there so we keep it p). Then he bought 4 more which mean +4, so the answer is p+4

The area of circular pound is 21.5 m2. Work out the diameter of the circle. Give your answer correct to the nearest centimetre.

Answers

The diameter of the circle is 5.2m.

The area of circular pond is 21.5 m2

area of a circle is given by the formula

area = πr²

21.5 = 3.14 r²

r² = 6.84

r = ±2.6

r is a length and it cannot be negative

So, r = 2.6

Diameter is twice of radius

d = 2r

d = 2(2.6)

d = 5.2

Therefore, the diameter of the circle is 5.2m.

To learn more about circle refer here

https://brainly.com/question/24375372

#SPJ1

Use the elimination method to solve the system of equations.3x + 4y = 8x - y = 12

Answers

Answer:

We have to use to elimination method to solve the provided system of equations, this method essentially involves eliminating one of the variables in the first equation by adding or subtracting a multiple of the second equation:

[tex]\begin{gathered} 3x+4y=8\rightarrow(1) \\ \\ x-y=12\rightarrow(2) \end{gathered}[/tex]

The elimination of the variable y in equation (1) is achieved by adding the 4 times the equation 2 into the equation (1), the steps of the process are as follows:

[tex]\begin{gathered} 3x+4y=8 \\ \\ + \\ \\ 4\times(x-y)=4\times12 \\ \\ ------------------------------- \\ \\ 3x+4x=8+48 \\ \\ \\ 7x=56 \\ \\ \\ x=8 \end{gathered}[/tex]

We have found the x value, the last step is to plug in the variable x value in any of the (1) or (2) equations, and solve for the y variable, the steps are as follows:

[tex]\begin{gathered} x-y=12 \\ \\ x=8 \\ \\ \therefore\Rightarrow \\ \\ 8-y=12\rightarrow y=8-12=-4 \\ \\ y=-4 \end{gathered}[/tex]

The final answer is as follows:

[tex](8,-4)\Rightarrow\text{ Option \lparen B\rparen}[/tex]

Therefore the answer is Option(B).

If ABCD is dilated by a factor of 2, thecoordinate of B' would be:543С2B1-8 -7-6-5-4-3-2-1 012.34567891011-1A-2D-3B' = ([?], [ ]=4Cotor

Answers

We have a figure ABCD that is dilated by a factor of 2.

We have to find the coordinate of the image point B'.

If the center of dilation is the origin (0,0) and the factor is k = 2, we can write the rule:

[tex](x,y)\longrightarrow(2x,2y)[/tex]

Then, for B(-1,1), we will get:

[tex]B=(-1,1)\longrightarrow B^{\prime}=(2\cdot(-1),2\cdot1)=(-2,2)[/tex]

Answer: B' = (-2,2)

laws exponents multiplication band power to a power simplifymake it small steps please the smallest you can like the minimum steps

Answers

Answer:

-64x^12

Explanation:

Given the expression

m = -4x^4

You are to look for m^3 as shown;

[tex]\begin{gathered} m=-4x^4 \\ m^3\text{ = (-4x}^4\text{)}^3 \\ m^3=(-4)^3\times(x^4)^3 \end{gathered}[/tex]

Open the bracket:

[tex]\begin{gathered} (-4)^3\text{ }\times(x^4)^3\text{ = (-4}\times-4\times-4\text{)}\times x^{^{12}} \\ (-4)^3\text{ }\times(x^4)^3\text{ = (16}\times-4\text{)}\times x^{^{12}} \\ (-4)^3\text{ }\times(x^4)^3=-64x^{12} \end{gathered}[/tex]

THe correct answer is -64x^12

Complete the similarity statement relating the three triangles in the diagram. Show work.

Answers

Answer:

[tex]\Delta JKM\approx\Delta MKL\approx\Delta JML[/tex]

Explanation:

There are three right angled triangles are in the diagram of three different sizes.

Identify the right angles.

In the first triangle, the right angle is angle M. In the second one, the right angle is angle K and in the third triangle, it is angle K.

Now, identify the shortest sides of each.

The shortest side in the first triangle is MJ, in the second triangle, it is JK and in the third triangle, it is MK.

In the given triangle JKM, notice that JK is the shortest side and angle K is the right angle. Write the other two tringles also in the similar form with the first two letters gives the short side and middle letter represents the right angle.

So, the similarity statement is

[tex]\Delta JKM\approx\Delta MKL\approx\Delta JML[/tex]

an arithmetic sequence has these properties: a1=2an = an-1 + 5 what are the first four terms of the sequence?

Answers

we have that the arithmetic sequence have:

[tex]\begin{gathered} a_1=2 \\ a_n=a_{n-1}+5 \end{gathered}[/tex]

so we can replace for n=2,3,and 4 to find the first four terms so:

for n=2

[tex]\begin{gathered} a_2=a_{2-1}+5 \\ a_2=a_1+5 \\ a_2=2+5 \\ a_2=7 \end{gathered}[/tex]

for n=3

[tex]\begin{gathered} a_3=a_{3-1}+5 \\ a_3=a_2+5 \\ a_3=7+5 \\ a_3=12 \end{gathered}[/tex]

and for n=4

[tex]\begin{gathered} a_4=a_{4-1}+5 \\ a_4=a_3+5 \\ a_4=12+5 \\ a_4=17 \end{gathered}[/tex]

So the first four numbers are

[tex]2,7,12,17[/tex]

Help

In 1846 the depth of the river was 5 feet deep.

In 1847 it dropped to 3.8 feet.

This year, 1848, it rose to 6.4 feet.

Find the percent change in river depth & complete the table.

Answers

The percent change in river depth is 52%

How to calculate the percentage?

From the information, the depth of the river was 5 feet deep, it dropped to 3.8 feet and in 1848, it rose to 6.4 feet. The new feet will be:

= -5 - 3.8 + 6.4

= 2.4

Therefore, the percentage change will be:

= New feet / Initial feet × 100

= (5 - 2.4) / 5 × 100

= 2.6 / 5 × 100

= 52%

Note that there was no table given.

Learn more about percentages on:

brainly.com/question/24304697

#SPJ1

What number is in the hundreds place after
simplifying?
(21 × 10¹)+(3×
X
10²) + (9 × 10²)
a) 4
b) 1
c) 3
d) 7

Answers

The digit in the hundreds place is 4

From the question, we have

(21 × 10¹)+(3×10²) + (9 × 10²)

=210+300+900

=1410

The digit in the hundreds place is 4 and the value is 400.

Multiplication:

Mathematicians use multiplication to calculate the product of two or more numbers. It is a fundamental operation in mathematics that is frequently utilized in everyday life. When we need to combine groups of similar sizes, we utilize multiplication. The fundamental concept of repeatedly adding the same number is represented by the process of multiplication. The results of multiplying two or more numbers are known as the product of those numbers, and the factors that are multiplied are referred to as the factors. Repeated addition of the same number is made easier by multiplying the numbers.

To learn more about multiplication visit: https://brainly.com/question/5992872

#SPJ1

Madeline earns $17,500 annually. What is the gross amount of hersemimonthly paycheck?a. $2,916.67b. $1,458.33c. $729.17d. $673.08

Answers

Given:

Annual earnings = $17,500

Asked: What is the gross amount of her semimonthly paycheck?

Solution:

To find the answer, we need to divide first the annual earnings by 12 to get the monthly paycheck. After that, we will divide it by 2 because we are looking for the semimonthly paycheck.

[tex]\begin{gathered} \frac{17,500}{12}=1,458.33 \\ \frac{1,458.33}{2}=729..17 \end{gathered}[/tex]

ANSWER: C. $729.17

help me out with this question..this is a k11..remember this is a practice question not a graded one

Answers

Each flight has a probability of 60% or 0.6 of being on time. This means that its complement, or the probability that the flight isn't on time is:

[tex]\text{\textasciitilde{}P(on time)}=1-0.6=0.4[/tex]

It is 40% or 0.4. "~P(on time)" stands for the probability of the flight not being on time.

1. The probability that at least 2 flights are on time is:

To find the probability that 2 or more flights are on time we can fight the probability that "0" or "1" are not on time.

[tex]P(0\text{ on time)}=0.4^9=0.000262144[/tex][tex]\begin{gathered} P(1\text{ on time})=\frac{9!}{1!\cdot(9-1)!}\cdot0.6\cdot(0.4)^8 \\ P(1\text{ on time)}=9\cdot0.6\cdot(0.4)^8=0.003538944 \end{gathered}[/tex][tex]\begin{gathered} P(1\text{ or less on time)}=P(0\text{ on time)}+P(1\text{ on time)} \\ P(1\text{ or less on time)}=0.000262144+0.003538944=0.003801088 \end{gathered}[/tex]

The probability of 2 or more flights are on time is:

[tex]P(2\text{ or more on time)}\cdot=1-0.003801088=0.996198912[/tex]

The probability of 2 flights or more are on time is 0.996198912

2.

We need to calculate the probabilities of 7,8 and 9 flights are on time and then subtract by 1.

[tex]\begin{gathered} P(7)=\frac{9!}{7!\cdot(9-7)!}\cdot0.6^7\cdot0.4^2 \\ P(7)=36\cdot0.6^7\cdot0.4^2=0.161243136 \end{gathered}[/tex][tex]\begin{gathered} P(8)=\frac{9!}{8!\cdot(9-8)!}\cdot0.6^8\cdot0.4 \\ P(8)=9\cdot0.6^8\cdot0.4=0.060466176 \end{gathered}[/tex][tex]P(9)=0.6^9=0.010077696[/tex]

The probability of at most 6 flights are on time is:

[tex]\begin{gathered} P(6\text{ or less on time) = 1 - (}P(7)+P(8)+P(9)) \\ P(6\text{ or less on time) = 1-(0.161243136+0.060466176+0.010077696)=}0.768212992 \end{gathered}[/tex]

The probability of 6 or less are on time is 0.768212992.

3.

The probability of exactly 5 flights are on time is:

[tex]\begin{gathered} P(5)=\frac{9!}{5!(9-5)!}0.6^5\cdot0.4^4 \\ P(5)=126\cdot0.6^5\cdot0.4^4=0.250822656 \end{gathered}[/tex]

The probability of exactly 5 flights are on time is 0.250822656.

Hello, I need help solving is and finding the checkpoints

Answers

Answer:

161 feet lower

166 feet

Explanations

a) Given the following parameters

Distance of checkpoint 3 above the sea = -197 feet

Distance of checkpoint 5 above the sea = -36 feet

Determine the distance between the checkpoints

Distance = checkpoint 5 - (checkpoint 3)

Distance = -36 - (-197)

Distance = -36 + 197

Distance = 161feet

Hence checkpoint 3 is 161 feet lower than checkpoint 5

b) If the top of the hill is 363feet above check point 3, the altitude of the top of the hill is given as:

Altitude = -197 + 363

Altitude = 166feet

Hence the altitude of the top of the hills is 166feet

point(10) On Myra's family vacation, her mom drove on the highway at aconstant speed of 60 miles per hour. Based on this rate, which of thefollowing times and miles driven are correct? Select all that apply. GRP.30190 miles driven in 3 hours150 miles driven in 2.5 hours30 miles driven in 30 minutes

Answers

[tex]\begin{gathered} \text{speed}=\frac{dis\tan ce}{\text{time}} \\ \text{speed}=60\text{ miles per hour} \\ \text{The possible speed = }\frac{150}{2.5}=60\text{ miles per hour} \\ 30\text{ miles per 30 minutes is also correct} \end{gathered}[/tex]

Solve. −45 = −2b + 3 + 8b

Answers

Answer:

b= -8

Step-by-step explanation:

just trust me dude. I know what I'm doing

Which expression is a factored form of −32m+96 −8(4m+12) 32(−m−3) 16(−2m+6) 32(−m+3)

Answers

Answer:

32(−m+3)

Step-by-step explanation:

1. Find the greatest common factor.
   32 is the GCF

2. Factor.
   32(-m + 3) -- you can check with distrubutive property.

find 3 consecutive odd integers when the sum of the first and third integer is 26

Answers

Let's call the first odd number x.

The next odd number will be x + 2, and the last one will be x + 4.

If the sum of the first number and the last one is 26, we have that:

[tex]\begin{gathered} x+x+4=26 \\ 2x+4=26 \\ 2x=22 \\ x=\frac{22}{2}=11 \end{gathered}[/tex]

So the numbers are:

[tex]\begin{gathered} x\to11 \\ x+2\to13 \\ x+4\to15 \end{gathered}[/tex]

11, 13 and 15.

There are 27 students in Mrs. Mello's class. fin the total number of pages they read by the end of November.

Answers

The total number of pages they read by the end of November is 2916 pages.

As the question, Chapter 1 has 35 pages, Chapter 2 has 38 pages and Chapter 3 has 35 pages. All these pages are read by all 27 students by the end of November.

⇒   Total number of pages in Three chapters  [tex]=35+38+35[/tex]

                                                                             [tex]=108[/tex]

⇒   Total number of pages in all three chapters read by one student = 108

⇒   Total number of pages read by 27 students  

                                      [tex]= total number of pages in three chapters * 27[/tex]

                                      [tex]=108*27[/tex]

                                      [tex]=2916[/tex]

Therefore, The total number of pages 27 students read by the end of November = 2916.

To know more about pages problems:

https://brainly.com/question/25996348

The full question is

There are 27 students in Mrs. Mello's class. fin the total number of pages they read by the end of November.

           

                                                   

Other Questions
Hello can you assist me with number 11Find the midpoint 13. What transformations take place by graphing the function below in respect to its parents gofunction? Check all that apply.Up 7 unitsDown 7 unitsLeft 7 unitsRight 7 units3f(x)=(x+7) +2Up 2 unitsDown 2 unitsLeft 2 unitsRight 2 unitsTheis)TextesVertical StretchVertical CompressionReflection in x-axisReflection in y-axis81 What is the slope of the line that passes through the points (10,8) and (7,14)? What is a student portfolio?O an official record of a single student's course gradesO a collection of a single student's best academic workO a document explaining assignments for a specific courseO a list of necessary courses to graduate with a specific degree Convert the measurement as indicated 83 qt = Gal Qt Jack has an old scooter. He wants to sell it for 60% off the current price. Themarket price is $130. What should his asking price be? Explain your reasoning. At a certain hospital 39080 patients had their falls reported in the winter of 2004, thisrose to 42045 patients in the winter of 2014 (Lifespan, 2019). How would you calculatethe percentage rise in patients from 2004 to 2014?In a several sentences, how would you apply this to your life or job? If you had to teachsomeone who did not know how to this, what would be the steps from beginning to endthat you would use to teach them so that they would eventually do it accurately as youwould?Professor,I would calculate the percentage rise in patients from 2004 to 2014 by For each value of v, determine whether it is a solution to v - 42 = 11 How many real sixth roots does -729 have? -6 = 3 - 4 = 4 + Solving systems of equations using substitution method 6ft 3ft 8ft 16ft area of irregular figures Translate the following word phrase to an algebraic expression and simplify: 8 times the difference of 6 times a number and 3 A company is going to make a storage container with sheet steel walls. The container will be in the shape of a rectangular prism, as shown below. If the sheet steel costs $30 for each square meter, how much will the sheet steel cost in total? I need help on homework 3.Which term best describes algae? The ages (in years) of the 5 doctors at a local clinic are the following30, 40, 39, 36, 30Assuming that these ages constitute an entire population, find the standard deviation of the population. Round your answer to two decimal places In the items below a physical property is identified along with two objects/figures. For each item identity whether the property applies to one, both, or neither of the objects/figures listed and explain why it does or does not.1) Property: MassCylinder Square Yusuf is going to the amusement park, where he has to pay a set price of admission and another price for tickets to go on each of the rides. The total amount of money Yusuf will spend is given by the equation y=4x+20y=4x+20, where yy represents the total amount of money, in dollars and cents, and xx represents the number of rides Yusuf goes on. What could the number 20 represent in the equation?A.The total amount of money Yusuf will spend if he goes on 1 ride.B.How much it cost for a ticket to go on one of the rides.C.The total amount of money Yusuf will spend if he goes on 0 rides.DThe total amount of money Yusuf will spend if he goes on 100 rides. What is imperialism?Question 8 options:A: The idea/policy of expanding the territory of a countryB: The idea/policy of remaining isolated from world affairsC: The idea/ policy of building up armed forces What is the value of x?2445X=(Simplify your answer. Type an exact answer, using radicals as needed.)